Получение фотонного пропагатора

Об этом уже спрашивали (см. Получение фотонного пропагатора ), но при выводе фотонного пропагатора, когда мы приходим к:

[ г мю ν к 2 + α 1 α к мю к ν ] Д ~ ν λ ( к ) "=" дельта λ мю

Мы должны инвертировать оператор слева, чтобы получить пропагатор. Я знаю, что мы должны использовать анзац:

Д ~ ν λ ( к ) "=" А г ν λ + Б к ν к λ

для определения коэффициентов А и В. Но разве нам не нужны два уравнения для определения двух переменных? Если да, то какое второе уравнение?

Также я не мог продвинуться очень далеко, используя это условие выше, все, что я получил, это:

3 А А + Б α "=" 4 к 2

Это вообще правильно? Если да, то как действовать дальше? Буду очень признателен за подробное решение.

Когда вы говорите, что об этом уже спрашивали, вы имеете в виду это: physics.stackexchange.com/questions/137577/… это хорошая идея, чтобы поместить ссылку, на которую вы ссылаетесь, в своем посте, спасибо
Не нужно извиняться за мои замечания, вам нужно извиниться только в том случае, если / когда вы действительно ошибетесь на личном уровне :), удачи с вашим вопросом.

Ответы (2)

Как автор этого вопроса, я думаю, что смогу помочь. Я буду использовать те же обозначения, что и в исходном вопросе.

Начиная с уравнения

(1) ( к 2 г мю ν + ( 1 1 ξ ) к мю к ν ) Д ν р ( к ) "=" я дельта мю р

мы делаем Анзац

(2) Д мю р ( к ) "=" А г мю р + Б к мю к р .

Подставив этот анзац в уравнение ( 1 ) мы получаем

(3) [ к 2 г мю ν + ( 1 1 ξ ) к мю к ν ] [ А г ν р + Б к ν к р ] "=" я дельта мю р .

(Вы упускаете фактор я с правой стороны ( 3 ) в вашем вопросе, но я поставлю его здесь.)

Что вам нужно сделать, не решить А и Б изолированно, но на самом деле сравните коэффициенты в обеих частях уравнения. ( 3 ) . Расширив продукт и после небольшой алгебры, вы должны прийти к

А "=" я к 2 , Б "=" я к 4 ( 1 ξ ) ,

который даст вам коэффициенты для обратного ( 2 ) .

Теперь я вижу, что понял это совершенно неправильно. Большое спасибо за подробный ответ.

Поскольку вы используете тензоры с греческими индексами, я хочу указать, что соглашение подразумевает, что Д ¯ ν λ ( к ) "=" А г ν λ + Б к ν к λ технически 16 уравнений, что более чем достаточно, чтобы выделить для А и Б . Если г ν λ является метрикой и к ν является неоператорным тензором импульса, то оба члена в правой части должны быть симметричными, что означает, что у вас есть не более 10 уравнений (чем меньше переопределенных, тем лучше).

Теоретически, вы должны быть в состоянии изолировать А и Б вводя различные компоненты г и к . Не зная, из чего состоят эти тензоры, я не могу помочь. Но я могу сказать вам, что это не тот случай, когда у меня слишком мало уравнений (на самом деле, я бы расстроился, если у меня слишком много уравнений).